Question in orthogonal trajectories

Click For Summary
SUMMARY

The discussion centers on the calculation of orthogonal trajectories for the family of curves defined by the equation g(x, y) = x³/y² = a. The user presents the differential equation 2ay/dx = 3x² and seeks validation of their answer. The correct approach involves calculating the gradients ∇f and ∇g, where ∇g = (3x²/y², -2x³/y³) and ∇f = (1, 2y). The condition for orthogonality is established by ensuring that ∇f.∇g = 0, leading to the equation 3x²/y² - 4x³/y² = 0.

PREREQUISITES
  • Understanding of differential equations and their applications.
  • Familiarity with the concept of orthogonal trajectories in calculus.
  • Knowledge of gradient vectors and their significance in multivariable calculus.
  • Proficiency in manipulating algebraic expressions involving polynomials.
NEXT STEPS
  • Study the method for finding orthogonal trajectories in detail.
  • Learn about gradient vectors and their applications in optimization problems.
  • Explore the implications of the condition ∇f.∇g = 0 in multivariable calculus.
  • Practice solving differential equations related to families of curves.
USEFUL FOR

Students and professionals in mathematics, particularly those studying calculus and differential equations, as well as educators looking to enhance their understanding of orthogonal trajectories.

manal950
Messages
177
Reaction score
0
Hi

290514745.jpg

answer b
466302518.jpg


I forget first step

2aydy/dx = 3x^2


please can check my answer
 
Physics news on Phys.org
I'm not able to follow the logic of your method, but it does not seem to give a valid answer. The original family is g(x, y) = x3/y2 = a; if your answer f(x, y) = y2+x = c is correct then you should find ∇f.∇g = 0. ∇g = (3x2/y2, -2x3/y3); ∇f = (1, 2y); ∇f.∇g = 3x2/y2 - 4x3/y2.
 
Question: A clock's minute hand has length 4 and its hour hand has length 3. What is the distance between the tips at the moment when it is increasing most rapidly?(Putnam Exam Question) Answer: Making assumption that both the hands moves at constant angular velocities, the answer is ## \sqrt{7} .## But don't you think this assumption is somewhat doubtful and wrong?

Similar threads

  • · Replies 2 ·
Replies
2
Views
1K
  • · Replies 2 ·
Replies
2
Views
3K
  • · Replies 7 ·
Replies
7
Views
1K
  • · Replies 6 ·
Replies
6
Views
2K
  • · Replies 25 ·
Replies
25
Views
2K
  • · Replies 1 ·
Replies
1
Views
1K
  • · Replies 9 ·
Replies
9
Views
2K
Replies
6
Views
1K
  • · Replies 2 ·
Replies
2
Views
5K
  • · Replies 5 ·
Replies
5
Views
2K